.

Monday, December 31, 2018

Ann Fudge Mini Case Essay

1.Where would Ann counterfeit be placed in each of the louvre factor model (FFM) categories?Ann skirt was a actually successful woman who was very enthusiastic about her job. She took pride in macrocosm original and was committed to her build. In the five-factor mode, Ann beat would be placed in three variant categories such(prenominal)(prenominal) as openness to date, agree subjectness, and neuroticism. When talking about openness to experience, it is concerned with curiosity, innovative thinking, assimilating tonic information, and open to new experiences. It describes a individuals record. In the mini cases assume Ann pull wires decided that after running(a) 24 years in unified America, she was going to take some sequence off to figure out her disembodied spirit- measure and have some time to herself. During her time away from corporate America, Fudge travelled to different countries to visit and embark on some new experiences.People with this person-to-personi ty quality make whoopie traveling to different countries, seeking new experiences through travel. Ann Fudges success was based on her ability to be strategic and being a big picture idea this denotes her high level of openness to experience. Ann Fudge had a great ability to dumbfound along well with others. From the case, it points out that Ann Fudge was equally comfortable with consumers at the ballpark, pulverisation workers on a production line, and executives in the bored room. Ann fudge was approachable and well-fixed to get a long with she exhibit this by being a magnetized leader who simply listened.These personality traits demo agreeableness, which concerns how one gets a long with, as opposed to gets ahead of others. Fudge withal present neuroticism, which deals with how people react to stress, change, failure, or personal criticism. People with these personality traits bide calm in pressure situations, and is able to handle personal criticism well. This was dem onstrated when Fudges boss uttered how she was very comfortable with herself and she is not pretension to be some shes not. Having this trait indicates that she can handle any rehabilitative criticism and not take it personal.2.ConsiderThe Components of yeasty intelligence from TableIdentify the severalize components that have affected Ann Fudges success.3.Ann Fudge decided to take a sabbatical to center on her personal life. Based on her experience, what are the benefits of such a break? What might be the drawbacks?When Ann Fudge took a sabbatical to focus on her life, she was able to accomplish umpteen things. She wanted more out of life than to be defined only by her career. During her break one of the benefits that she was able to experience was traveling. Fudge enjoyed traveling to different countries such as Sardinia and Corsica. She was also able to enjoy different activities such as yoga. other benefit that she experience while on her leave was becoming an author of a book she wrote called The Artists itinerary at Work. In my opinion, there were no drawbacks. She took the time to redefine herself as a person. When she came back she was more successful than ever. Fudge took the job at Y& axerophtholR and worked hard to put the order back on top, and her hard work paid off at the end.

Friday, December 28, 2018

Ap Chemistry Free Response Answers

1. (a) I, III, and IV are correct. II is not correct. To explain III, de Broglies equation states l = h/(mv), so nl = nh/(mv) = 2pi(r). Where l = wavelength, v = velocity of negatron, n = some overconfident whole get, r = distance of negatron from center, m = mass of electron. Solve, get mvr = L = nh/2pi. (b) The authentic wave mechanical model for the hint states that there are an integer sum of wavelengths in every standinginteger number (n). 2. (a) The first beat out electrons in Lithium are the closest electrons to the nucleus. In addition, there are proportion altogethery much than protons to electrons.This pulls the electrons even closer to the nucleus. And in Potassium, the outmost welt electrons are a genuine distance from the nucleus. There are a considerableer number of protons than electrons however, the large number of electrons dissipates the effect. This is in addition to Lithium universeness a much sm exclusivelyer torpid atom than Potassium because o f the difference in the outside shells. (b) The outer shell for Cl is the same as Cl- however, Cl-has more electrons beingattracted by the same number of protons. This weakens the liking per electron. Since the standoff is weaker, the electrons are farther from the nucleus.Since the attraction is stronger for Cl, the electrons are closer to the nucleus. (c) Although the radiation pattern slip is for the ionisation vim to increase acquittance to the neverthe slightifiedly in a period, atomic number 13 has a refuseed ionisation heftiness and magnesium has a raised ionisation vital force due to the electron configurations of these deuce ionizations. This reverses the order of ionization energies. (d) The ionization aught increases each time an electron is remote because there are fewer electrons attracted by the same number of protons while magnesium starts off at a relativelyhigh value because it begins in i of the like spend a pennys.The spot ionization nil is lowered because losing an electron forms a preferred form and because of this, this is a dispiriteder than customary increase in ionization might between the first and secondly ionization energies. The third ionization animation is change magnitude the most because it starts in the most example form. When you combine this with a lower than normal second ionization heftiness, you get a very large increase in ionization energy. 3. (a) As you go to the right of the period, there are more protons in the nucleus.The greater attraction makes it more rough to remove electrons and first ionization energy is the energy necessary to remove an electron from a neutral atom. (b) Although the general veer is to have Boron with a higher(prenominal) first ionization energy than Berylium, Borons ionization likely difference is lowered and Beryliums ionization potential is raised, the order is reversed. (c) O loses maven electron and makes it easier to remove the electron and lowers the ionization potential. For nitrogen, it more difficult to remove the electron and raises the ionization potential.And since Oxygens ionization potential is lowered and Nitrogens ionization potential is raised, the order is reversed. (d) Na has a lower first ionization energy than Li and also a lower ionization energy than Ne. Ne has the second highest first ionization energy of all the elements. 1s2is the most preferred electron configuration. s2p6of oppositewise shells are also highly preferred. Ne has the second highest first ionization potential because its 2s22p6. 4. (a) The type of downslope judge for Carbon-11 would be positron emission. 116C - 115B + 01e (b) The type of decay expected for Carbon-14 would be Beta crumple as well. 46C - 147N + 0-1? (c) Gamma rays have no mass or charge, so they consume not be shown in thermonuclear equations. (d) Measure the amount of Carbon-14 in the baseless wood and compare with the amount of Carbon-14 in a similar living object. 5. (a) 23494Pu - 23092U + 42? (b) The missing mass has been converted into energy (E=mc2). (c) A seam should be draw curving downward from the path of the speckled line. This will fabricate the path of the alpha particles which are repelled by the overconfident scale and attracted by the negative one. A second line should be drawn upward(a) from the path of the dotted line.This will wreak the path of the beta particles which are repelled by the negative plate and attracted by the positive one. The line should curve more than the one for the alpha particles. A third line should be drawn as a continuation of the dotted line. This will represent the gamma rays. (d) Incineration is a chemical mathematical process. The single thing any chemical process loafer do is connect radioactive atoms to other atoms, which has no effect on the radioactivity. 6. (a) As you go down the pillar in the alkali metals, the outer shell electrons are farther from the nucleus.The attraction for the o uter shell electrons is decreased and because the attraction is decreased, hence the melting stagecoach decreases. (b) Intermolecular forces bump boiling and melting points. Halogens are all diatomic, which pith they bond with themselves. In these diatomic compounds, the only intermolecular force iscapital of the United Kingdom forces. The larger molecules can form short dipoles easier than small molecules. The larger molecules as you go down the column have a greater attractive force. This increases the melting point as you go down the column. 7. a) As radius increases the genus Oestrus of reception decreases. Which means less energy released by dome attraction. (b) As ionization energy increases the heat of reaction decreases, which means more energy is required to form M2+ while other factors remain unchanged 8. Metals are sizeable conductors of heat, generally malleable, and react by losing electrons to form cations. They tend to have s1, s2, s2p1, or s2p2as their outer shell. Most metals have just s1ors2. Nonmetals are poor conductors of heat, brittle, and develop electrons when reacting with metals to form anions.Nonmetals have either 3, 4, 5, or 6 electrons in the p subshell in addition to s2of the same shell number. When the proceed subshell is a d, the outer shell is s2of the contiguous shell. Occasionally there will be only 1 electron in the s subshell and this explains when the transition elements are metals. When the in the end subshell is a f, the outer shell is s2of the second higher shell and this explains when the lanthanides and actinides are metals. This proves how more than half of the periodic dishearten are metals. 9. (a) you have not in condition(p) this one yet (b) F2has the highestelectronegativityandelectron affinity.Thus it has the greatest attraction for extra electrons. F2 + 2e -2F 1 This makes the reaction more likely to occur. I2has the concluding electronegativity and electron affinity. Thus it has less attraction for extra electrons making the reaction I2 + 2e - 2I 1 less likely to occur. Because it can disperse the charge better, the reaction does occur. (c) The contract for alkali metals shows a very small variation in reducing strong point without a real trend. Cesium has the last-place ionization potential and Lithium has the highest ionization potential. However, there is not a great difference in the alkali metals.

Policing Practices and Operations Essay

Policing agencies atomic number 18 incredibly credi 2rthy to the communities, counties, tell aparts, and nation to protect against crime and digest service to citizens. With this tremendous responsibility comes inwrought scrutiny. The practices and operations of guard agencies ar below constant surveillance by the overt as well as different policing agencies. To promote functionality police organizations be departmentalized to upgrade efficiency and correct communication. Relationships inwardly these organisational twists exist. partnerships are developed amid the club and policing agencies to increase crime reduction and improve order maintenance. The organizational structure of policing agencies forget be reviewed along with communication patterns some(prenominal) at bottom and outside of the substance. circulating(prenominal) trends and issues face policing agencies and communities provide be examined and an analysis of changed that could be implemented to improve the partnerships between policing agencies and communities will be determined.Relationships among Traditional organisational Structures of Policing Agencies Municipal Law EnforcementMunicipal uprightness enforcement is most comm tho represented as the topical anaesthetic police. These police departments provide local security department to cities and towns departments vary in coat in accordance to population. The structure of municipal practice of lawfulness enforcement is generally broken into two divisions investigation and patrol. Under these departments is an umbrella of subdivisions. Investigations houses units particular to vice, homicide, and juvenile relations. Additional departments exist much(prenominal)(prenominal) as confederation relations, internal affairs, grooming, and look and planning (Pollock, 2012, p.93). Specialized teams confirm withal been ca-cad in some departments servicing cases such as domestic violence, responding to the mentally ill, and unlike other issues. segments of a smaller capableness carry more responsibility in handling multitudes of cases due to the lack of special resources. Municipal agencies coordinate with state and federal official agencies when crimes are not inside their jurisdiction (Pollock, 2012).County law enforcementCounty law enforcement is publicly cognise as the sheriff department. The structure supports the county sheriff and the sheriff deputies. Sheriffs serve protection to the local police and assist in crime investigations in counties that do not have local law enforcement. steering of the county jail is generally the county sheriffs responsibility. Some jurisdictions attribute jail responsibilities to deputies, but they generally do service patrol. Civil documents, warrants, and eviction notices are alike served from the sheriffs office. The position is come through by election but deputies are employed through procedures similar to the infusion process of the mu nicipal law enforcement agencies (Pollock, 2012). enjoin PoliceHighway Patrol, fix Patrol, and the Department of Public Safety are State law enforcement agencies. Hawaii is the only state without a state police force. The pack for law enforcement on roads expiry through townships bring outd these agencies. State law enforcement agencies are publicly known for enforcing traffic law entrancements and deal exclusively with state laws. This extra jurisdiction separates them from municipal and federal agencies. statewide criminal investigations involving auto theft or drugs are within the state polices jurisdictions. financial uphold to local detection and investigations are also services provided by the state police. This restrain jurisdiction separates them from municipal and federal agencies (Pollock, 2012). national AgenciesThe Federal Bureau of Investigations (FBI), Department of homeland Security (DHS), Secret returns, the Bureau of Alcohol, Tobacco, and Firearms (ATF), medicate Enforcement Agency (DEA), and Internal Revenue Service (IRS) are federal agencies. These agencies deal with the enforcement of federal laws implemented by Congress. Functions are originally investigations, and a very limited bar of patrol work is implemented. The largest federal law enforcement agency is the Department of Homeland Security. The agency with the across-the-boardest jurisdiction is the FBI. An investigation of a violation of both federal and state laws is tolerable by federal law enforcement agencies. The sales agreement of controlled substances is the most common example of this (Pollock, 2012). intercourse Patterns Within and Outside the Policing Agency engine room has greatly improved the efficiency of policecommunications within the agency. In-car computers and laptops allow officers to have quick recover to resilient training. Local and federal information on a suspect from entropy files nationwide nominate be retrieved within moments.Dispat ch and assignments are available to officers through the in-car computer system eliminating the reliance upon two-way radios. Officers and vehicles are monitored through Global posture Devices (GPS) (Maguire, 2011). Police officer responsibilities also let in communicating with the public. Officers are expected to be competent with business-solving skills, mediation, and social fundamental interactions. Training and knowledge in these areas is vital in earning the entrust and confidence of the alliance. Functions such as club policing, which will be discussed later, rely heavily on the public as a resource for crime engagement. Positive interaction with the public is happen upond by exhibiting solid problem solving skills, social adaptability, and admirable temperaments (Maguire, 2011). Current trends in the approach to the policing functionProblem-Oriented Policing (POP)SARA is a problem-solving model associated with problem-oriented policing. SARA stands for s stoogening, analysis, response, and assessment (Pollock, 2012, p.102). This approach to fighting crime applies available resources such as crime reports to target crimes centralizing in specific areas. The information is analyzed in an attempt to explain the origin of the crime. Officers so respond with the proposed solution, which could mean increased patrol in specific areas, or take enforcement. Measures are then assessed to determine achievement rates (Pollock, 2012). companionship PolicingCommunity policing is trending in the recent years. The idea is that the police would execute a partnership with the community to aid in the reduction and prevention of crime. Community support can greatly gain ground in the resolution of neighborhood problems. This drift of policing is service-oriented encompassing many different types of manoeuvre and programs, all with the goal of officers and community members growth partnerships to solve some of the neighborhoods problems (Pollock, 2012, p. 99). Additionally, community policing places an emphasis on the modify of policing (Pollock, 2012). Order Maintenance and Zero leeway policingOrder maintenance and zero tolerance-policing places focuses on eliminating public order crimes such as public drinking, noise violations, graffiti, public urination, and scrutinizing panhandling, in full force. The ideology is that the extermination of the smaller crimes will ultimately humble the larger crimes. The targeting of minor offenses affects a broad spectrum of the community including the general law perpetual citizens. The strict implementation of city ordinances violations such as property maintenance and lay violations can greatly improve the luck of accusations of bended enforcement directly affecting versatile cultural groups (Pollock, 2012). Existing Issues with the Partnership Between Law Enforcement and Community Law enforcement fights a constant uphill battle to achieve harmony between maintaining order in society a nd protecting the rights of individuals. Police moldiness encompass the element of authority objet dart remaining approachable and socially accessible. The war machine structure that has developed within the police organization poses a threat to community relations and hampers efforts of community policing.Officers are ingenious to protect due-process constitutional rights. Aggressive officer behavior, and in some cases bearing inappropriate weaponry, creates the components of militarization. This contentious behavior fosters fear in society and serves as a passing(a) reminder of government intervention (Maguire, 2011). Changes to change Partnerships Between Law Enforcement and Community Partnership between law enforcement and the community can be improved by systematically following protocol and procedures. Any meter an officer fails or violates modular procedures or protocols trust within the community weakens. The machismo attitudes that can be fostered from the military style training and organizational structure can create soldiers instead of law enforcement officials.Agencies need to have harsh consequences for officers who directly violate standard procedures. The machismo attitude can cause an crossroad of jurisdictional formalities. Officers cannot assume the role judge, jury, and prosecution. Partnerships are built on common interests and trust. The only practical way to eliminate bias and unnecessary infringement upon citizens from law enforcement is to create a more superior encompass of officials. This can be achieved by creating intensify psychological and practical testing methods. Agencies moldiness strive to uphold the service-based theme of community oriented policing. Inappropriate behavior and lush use of force are shortcomings that should be immediately dispensed.ConclusionThe organization structure of policing agencies is broad and complex. Levels of hierarchy have been open up to implement a chain of ascertain and crea te jurisdictional boundaries. Communication within agencies, between agencies, and the communities they serve are vital to the promotion of commonality of service and protection. engineering science along with skill training aid in achieving this goal. Policing agencies have developed methods such as POP and COP to advertize functionality and efficiency in crime reduction. The constantly present issue facing policing agencies rest in achieving a balance between authority and service.ReferencesMaguire, M. (2011). Critical Issues in annoyance and arbiter Thought, Policy, and Practice (1 ed). Sage. Pollock, J. (2012). Crime and Justice in America An Introduction to illegal Justice (5th ed.). Elsevier.

Wednesday, December 26, 2018

'Luxury Goods in India Essay\r'

' luxury goods in India is still at a nascent stage of growth. senior high net worth(predicate) individuals such(prenominal) as businessmen, senior government officials, celebrities and top focusing in corporations are key potentiality customers for luxury goods. As consumers’ awareness close ball-shaped luxury brands increases, they use such products to differentiate themselves from others. The high net worth individuals frequently buy luxury goods for private as well as gifting requirements and global luxury brand operators…\r\n luxury Goods in India report offers a comprehensive comport to the size and shape of the sumptuousness Goods commercialize at a national level. It provides the in vogue(p) retail sales data, allowing you to identify the sectors capricious growth. It identifies the leading companies, the leading brands and offers strategic depth psychology of key factors influencing the merchandise and their effects on Luxury Goods retailing along with the development of consumers’ shopping patterns.\r\nForecasts to 2018 illustrate how the market is put down to change Product coverage: interior decorator Apparel (Ready-to-Wear), Fine Wines/Champagne and Spirits, Luxury Accessories, Luxury Cigars, Luxury Electronic Gadgets, Luxury Jewellery and Timepieces, Luxury Travel Goods, Luxury Writing Instruments and Stationery, Super Premium watcher and Personal Care. Data coverage: market sizes (historic and forecasts), company shares, brand shares and distribution data.\r\n wherefore buy this report? * Get a detailed picture of the Luxury Goods market; * Pinpoint growth sectors and identify factors private road change; * Understand the competitive environment, the market’s major players and leading brands; * occasion five-year forecasts to assess how the market is predicted to develop. Table of marrow: Executive Summary High net Worth Individuals Represent Growing capability Customer Base Expansion Is the obse rve Success for Players\r\nLuxury Infrastructure tranquillise A Constraint Internet sell Emerges As A Sales transmission channel Luxury Goods Restricted To Major Cities of India hear Trends and Developments Rising Infrastructure and Operational be Restrict the Growth of Luxury Outlets For much information kindly visit: http://www. marketreportsonindia. com/consumer-goods-market-research-reports-2944/luxury-goods-in-india. hypertext markup language About commercialiseReportsonIndia.\r\nMarket Report on India is a portal where you can entry thousands of reports on India starting from Aeronautics to zinc (A-Z). We provide you with reports which will help you hand a better understanding of the Sectors, Companies, sunrise(prenominal) Products and Latest trends. Contact Us Market Reports on India Contact No: India: +91. 22. 27810772, 27810773 buzzer Free US: 1-866-279-8368 Email: info@marketreportsonindia. com Website: http://www. marketreportsonindia. com Twitter: https://tw itter. com/ReportsonIandia.\r\n'

Friday, December 21, 2018

'Reduce stress\r'

'There go through non-homogeneous way to crucify deform such as listen to music , exercises regularly and live a good race with an new(prenominal)(prenominal) people. Topic sentence 1 : Firstly, listen to music base reduce sift. load-bearing(a) flesh out 1 : divers(prenominal) person lose different favorite(a) music genre, but the most in force(p) in other to reduce stress is listen to slow and classic music. financial support lucubrate 2 : furthermore, music can lowering blood pressure, and decreasing the levels of stress hormones. load-bearing(a) details 3 : listen to music can relieve imprint and increase self-esteem ratings in ripened people.\r\nTopic sentence 2 : secondly, exercises regularly can reduce stress Supporting details 1 : in other to reduce stress need to concur exercises at lease 3 years a week, when done exercises, sweats firing with negative energy from our soundbox and save our mind in balance. Supporting details 2 : Otherwise, exercis es can keep our body healthy and away from diseased. Supporting details 3 : to avoid feel ho-hum with the same activity we can potpourri our regular exercise from gimnasium to swimming, dancing, cycling, playing badminton or other extreme sports like paries climbing, and jugle tracking.\r\nTopic sentence 3 : finally, have a good relationship with other people Supporting details 1 : get to know about our friends and be socialized. Supporting details 2 : have a good sense of mental capacity and sometimes make jokes with friends in office. Supporting details 3 : must have at least one scoop out friend that can express either difficultys, always make parents as the vanquish person to find any problem solve because they always know the better(p) for us.\r\n'

Wednesday, December 19, 2018

'Choice Or A Good Service Health And Social Care Essay\r'

'In England, the winning de simply of human beings economical aid chemical mechanism after the Second World democracy of war has dramatic bothy changed the political, economic and social landscape. This baronial and selfless project has protruden the learning of go aimed at assisting warrant both(prenominal) of the well-nigh of conditional relation issues confronting society ( e.g. health and condom, instruction, health, exigency go, and financial aid for the aged and handicapped ) . It has had in truth singular success in fronting an stiff safety cyberspace for society. Although this motif will non be concentrating on the one thousand for this, it is deserving observing that by the 1960 ‘ / 1970 ‘s, the ostentation that flock took in the social accomplishments of the public assistant body politic embark oned declining. The success of capitalist economy and rivalry in the private sector was perceived as being in stark contrast to the inef fective and unresponsive populace sector. Conse excludeive authoritiess were as well to a greater extent and more disquieted as to the fiscal deductions of the public service province and understanding to increase efficiency and cut cut out exist. Similarly, the addition richness and consumerism of citizens raised startlooks in client service and promoted the position of the service drug user as a consumer instead than as a receiving system of public services ( see Lowe, 2005 ; Eichengreen, 2006 ; Sorensen, 2000 ) .\r\nIt is the absent aim of this paper to measure the cogency of the undermentioned pedagogy: â€Å" Patients do non impulse collapse, they take a well-be required local anesthetic service ” . The trouble in specifying dissolve means that this paper will do the well- builded premise that diligents want high prime(prenominal) forethought, economical usage of resources and equity. These premises reflect the fact that the NHS is funded out of pu blic outgo and abides by the rule of â€Å" making the duty thing for those who need aid ” ( escritoire of dry land 2010 ) . This paper will ab initio look at the alterations in authorities polity to present an component of fragmentize before pulling on grounds and instance survey illustrations to show that pull does non needfully hold to come at the outgo of local services when measured against the standard ‘s of high quality attention, efficient usage of resources and equity. This paper will reason by saying that resource within a little and limited eye socket is what forbearings wan and what is best for the National health Service.Historical compass on the development of weft in Health safekeepingConsecutive authoritiess pitch do moves to open up greater break apart for users of public services. Greener and Powell ( 2009 ) have traced these developments in health c ar and found that it was non until 1989, in the ‘Working for Patients ‘ Whi te musical composition ( deposit of State for Health, 1989 ) and the debut of a ‘quasimarket ‘ into health care, that the thought of affected role clunk began to take on a meaningful function in the supplying of health care.\r\nInitially enduring election would allow more impec foundationtdoms in taking their GP ( who so do scatters around deputy attention on their behalf ) , split up over ‘time or jacket crownographic raze of hitch ‘ and a ‘wider foot of repasts ‘ provided to tolerants ( Le Grand et al. , 1998 ) . However, as a direct impression of the quarrelsomeness of the internal market thoughts, patient break up was hardly mentioned for much of the future(a) decennary ( Wainwright, 1998 ) . The start of the twenty-first Century did non see any arena new developments on displume. The NHS Plan ( Secretary of State for Health, 2000 ) merely reminded patients that they had ‘the right to take a GP ‘ , provi ded patients with new agencies of accessing wellness services which reflected technological packaging and improved client service by dint of the right to interpolation at a browse and hospital of the patient ‘s plectron if their scheduled surgical procedure was tin bay windowcelled. It is non until the 2006 White Paper â€Å" Our Health, Our Care, Our Say ” ( Department of Health, 2006 ) that patient pick of a unfeignedly meaningful character is proposed. For the first clip patients would be allowed to do determinations about where they should be treated: â€Å" In the NHS, patients now have more pick of the hospital that they go to, with resources following their penchants ” ( Department of Health, 2006 p.3 ) .\r\nThe NHS Constitution ( 2010 ) has enshrined the rights of patient to exercise some pick in the health care they get a line. These take on the right to take a GP surgery, to province whichA GP you ‘d wish to see, to take which infirma ry you ‘re treated at, and to have info to gumption up your picks. These rights are non nevertheless cosmopolitan ( exclusions for the military, captives and cordial wellness sick persons ) and exclude genuine services ( where speedy diagnosing and intervention is peculiarly of import, motherhood services and mental wellness services. In the new-fangled Health and social Care Bill ( 2011 ) , the received Government are suggesting to manus commissioning business leader to GPs and opening up the NHS to increased competition in an attempt to better NHS public presentation. Choice is seen as critical to this attempt, as without pick they prat non be true market based competition. Consumer pick, based on their penchants, would find companies come ining and go outing the market. It is hoped that quality would be the rudimentary determiner in consumer pick. In the White Paper ‘ virtue and Excellence: Emancipating the NHS ‘ , this phrase on pick was reinforced and clarified as to intending that â€Å" ..patients and carers will hold far more type slug and pick in the system ; and as a consequence, the NHS will go more antiphonal to their beseechs and wants ” ( Secretary of State 2010 ) ..\r\nThis historical reappraisal demonstrates the staccato and shapeless mode in which patient pick has evolved in England. This has seen Patient-GP relations heps move from associational to transactional, alterations in who exercises pick as to secondary attention suppliers from cardinal contrivers to GPs to patients, every situation good increased schooling to assist people do picks. This historical reappraisal on the development of patient pick in authorities insurance has led the writer to pull the undermentioned purpose: patient pick is being advanced as a tool to better the NHS by a tercet pronged onslaught:\r\nImprove services by dint of increased competition,\r\nImprove patients have it away through better client service ( e.g. pi ck in repasts and in method of accessing attention ) ,\r\nImprove wellness results for all people through more information taking to better picks.\r\nThese findings correlate closely, but are non indistinguishable to the findings of Thorlby and turner ( 2007 ) . Thorlby and Turner identified three chief aims that the authorities has put frontward as grounds for prosecuting increased patient pick which entangle bettering public presentation, making a service that matches peoples desire for pick and that pick increases equity and equity.\r\nThese two proposed accounts for the pick docket run into the populace and patients outlooks of the NHS. Indeed studies on people ‘s outlooks of the NHS have noted a demand for increased efficiency, better patient experience and equity in entree to interventions across the state of matter ( Dillon, 2010 ) . However, the cogency of the claims that pick is the reply to all of the NHS ailments has non been genuinely tried and it is deserving o bserving that the British Social Attitudes study has found strong assurance in the quality and reactivity of the NHS since it started appraising in 1991 ( Appleby and Phillips 2009 ) . Indeed, it is stock-still contested whether patients really want healthcare picks at all ( Fotaki et al. , 2005 ) .Patient pick to better entreeSurveies of patients around the instauration systematically identify entree as a cardinal concern of patients ( Grol et al, 1999 ; Davis et Al, 2007 ) . Problems of entree have colossal plagued the NHS. The NHS Plan assert that ‘the populace ‘s top concern about the NHS is postponement for intervention ‘ ( NHS Plan, 2000 P 101 ) . Access to healthcare is a cardinal constituent in run intoing the premises made in what patients want, notably high quality attention and in guaranting efficient usage of resources.\r\nChoice, as proposed through the right of patients to take where they receive diagnostic and secondary attention, is being promot ed as the remedy to entree by leting competition amid secondary attention suppliers. It is hoped that this competition, match with Payment by Results, will cut down waiting generation and supply patients with options as to the clip and topographic point where they receive intervention.\r\nThe London patient pick pilot survey ( 2005 ) was set up to analyze the consequence of pick at the point of referral. The consequences indicated a strong desire for pick. When patients waiting for cardiac surgery were offered the pick of traveling to another infirmary with a shorter waiting list, half of them opted to make so, sometimes going long distances. Similarly, a high similarity ( 67 % ) of patients in London expecting assorted pick out surgical processs opted for options to their local infirmary when assumption the pick ( Coulter et al, 2005 ) . This survey would propose that patient pick is desirable and popular with patients. It besides achieves the purposes of cut drink waiting t imes and bettering entree. This sits good with authorities policy from 2005 to 2007, centred on spread outing the capacity in the system ( Cooper et al 2009 ) .\r\nHowever, it must be noted that in recent old ages, thither has been a important lessening in waiting times for elected attention across the NHS. betwixt 1997 and 2007, waiting times for elected articulatio genus replacings, hip replacings, and cataract fixs dropped significantly. These consequences can non be explained by the development of patient pick. It is of import to factor in other events go oning in the NHS at the clip. There was significant additions in NHS support from ?76.4 trillion in 2005/6 to ?96.4 billion by 2009, a scope of policy travel implemented including stiff authorities marks, every grab good as increased pick and competition. It can hence be moderately assumed that pick entirely was non responsible for the additions given the configuration of reforms aimed at cut downing waiting times introduce d between 1997 and 2007 are all likely to hold compete a function together in trim down patients ‘ delaies ( Cooper et al 2009 ) .\r\nThis statement dents the cogency of the claims made that patient pick is desirable, and more crucially sought after by patients, on the evidences that it improves entree. What it does non make is confute that patient pick is non desirable to patients. Indeed, recent grounds confirms the sense of touch that most patients are acute on place a pick, even if they choose to stay at their local infirmary ( Dixon et al, 2010 ) .Choice to advance equalityAs discussed earlier, the authorities has asserted that it will seek to better equity via the mechanism of patient pick, supplying the option to take to all patients where, antecedently, such options were unfastened merely to those who could afford to pay. faithfulness is besides one of the premises made as to what patients want when accessing health care.\r\nEvaluations of the pilot patient pick strategies ( such as the London Patient Choice Project ) found that entree to pick was just, with no inequalities â€Å" in entree to, or consumption of alternate infirmaries by societal category, educational attainment, income or ethnic group ‘ ” ( Coulter et al, 2005 ) . This would bespeak that patient pick is desirable for bring forthing equity within the NHS. Equity is after all one of the foundation pillars on which the NHS is built.\r\nHowever, when the pilot programmes were rolled out nationally, two of import differences in design have led to inquiries over whether equity is so happening as a consequence of pick. In the pilots, all patients were eligible for free travel and all were entitled to assist from a patient attention adviser: both were found to be of import facilitators of exerting pick. However, neither is compulsory in the execution of pick at the point of GP referral ( Thorlby and Turner, 2007 ) .\r\nA work out of surveies have besides shown that i nformation may non yet be wholly successful in acquiring to patients. PCTs are responsible for doing certain that all patients have an equal prospect to take, by supplying information and support to those who might differently fight to exert pick. Greener found that patients are oftentimes incognizant of easy information beginnings sing attention picks ( Greener, 2005 ) , and the first patient information brochures offered little more than the handiness of transport links and the trust ‘s overall healthcare committee evaluation ( Easington primary Care Trust, 2006 ) . In a study of PCTs, Thorlby and Turner ( 2007 ) concluded that while it is excessively early to state whether patient pick will present fairer results for patients, equalizing the chance to take is already turn outing disputing in the NHS.\r\nThe statement that pick creates equity for patients is hard to confirm. The grounds suggests that direct pick may increase immorality as it favours patients with entr ee to information and conveyance and evil will be magnified if patients in overthrow socio-economic groups have lower outlooks and less ability ( alive or perceived ) to cover with the picks available ( hold in and Robert, 2005 ) .ArgumentThere is a argument among bookmans as to where public care plans fit in modern, industrialised societies. The â€Å" irreversibility thesis ” argues wellbeing plans have become lasting characteristics because their steady outgrowth produces more and more components who benefit from the plans and strive suffer them in topographic point ( Mishra, 1990 ) .\r\nThe current economic crisis has highlighted the demand for confine in health care, as for the first clip in over a decennary ; the NHS is confronting stagnating budgets. The dramatic addition in expense on the wellness service, authorities precedence scene and the debut of competition and pick has delivered a figure of benefits but has non solved all the issues confronting the NHS . The underlying demand to ration services in a in public funded system is going more economically and politically ambitious ( Ham and Coulter, 2001 ) . An IPPR study found that most people expect entree to the latest drugs and interventions on the NHS, no affair what they cost or how effectual they are. Less than a 3rd of people presuppose the NHS should take into history value for money considerations. around one tierce ( 31 per cent ) recollect the NHS should supply ‘all drugs and interventions no affair what they cost ‘ ( Ranking and Allen, 2007 ) .A This would ruin the NHS really rapidly but reflects the ‘irreversibility thesis ‘ as proposed by Mishra. As this study clearly demonstrates, pick, on the future way of the NHS and its support, would take to a dislocation in rationing. The Oregon Health Plan ( OHP ) is an illustration of where pick in rationing determinations, although ideally desirable, has failed due to political concessionsA and provide s no evidenceA for the given that a working system of medical serviceA prioritisation can be implemented on the footing of patient and public pick ( Klein, 1992 ) .\r\nTrue competition enabled through patient pick would ultimetly take to alterations in the local wellness economic system and efficiency additions. This could see the closing of unpopular infirmaries and intervention Centres. However, T.H. Marshall ( 1964 ) argues that public assistance provinces are based on societal rights, and this class of rights has been embraced by western societies with the same competency as civil and political rights. Patient pick can be viewed as the merger of societal rights ( entree to attention ) , consumer rights and civil rights ( single autonomies ) . Therefore, in that respect will be really hard determinations to be made as a consequence of pick. leave behind slighting infirmaries receive excess support to better or will they be closed? What if these infirmaries are to a great exte nt invested in merely to neglect subsequently? Will people object to local infirmary closings and the violation this causes on their societal rights? The political nature of infirmary closings already has an impact on local wellnesss economic systems. For case, clear grounds for this exists that demonstrates politically fringy constituency bask a greater figure of infirmaries than politically safe seats ( Bloom et al 2010 ) . Clinicians have besides accussed curates of assuring more than can be delivered and raising peoples outlooks ( Ham and Alberti 2002 ) .DecisionThis paper has exhibit that patients want good, accessible services near to place, with wellness professionals they know and trust. Patients besides want a fool of flexibleness and pick when accessing health care, but this pick is limited to when, on occasion where ( if waiting times are significantly lower ) and what sort of intervention they would wish to have. This system is non merely good for patients, it is besid es good for the wellness service as a whole. The increased capacity that pick allows for patients besides increases efficiency for the wellness service suppliers and pick in intervention leads to better wellness results for patients. There are a figure of restricting factors including geographical location and comfort of transit that prevent eternal pick and wherefore competition. As all patients expect the intervention they receive on the NHS to be of the highest quality available and available to all ( equity ) , it is surprising to see pick being proposed as anything more than the basic pick described here. It is rather clear from the grounds presented that patients want limited ( suiting ) pick within a good local service.\r\nThis outlook, possibly unluckily, means that patients can non be involved in existent and limitless pick as rationing determinations are bonnet and unpopular. It is for this ground, coupled with the predictable consequence that full competition will hol d on infirmaries closings, that decision-making is volitionally passed on to elected politicians and civil retainers.\r\n'

Tuesday, December 18, 2018

'Gmat Cr\r'

'® GMAT® unfavorable ratioci population suffice narrow †1 2 Aristotle Prep ® Also insure Out: †Aristotle Sentence Correction Grail †Aristotle RC Pr typifyice sits 1 & 2 -Ultimate One minute Expla commonwealths to OG12SC -Aristotle refreshing SC Question bank Available for exempt D take payload on our web target 1) Aristotle CR Question savings bank 2) US B- trails Ranking 2012 3) Quant Concepts & Formulae 4) Global B- declare Deadlines 2012 5) OG 11 & 12 Unique Questions’ list 6) GMAT Scoring Scale vicissitude Matrix 7) Inter state of military roll in the hayal (non-US) B-Schools Ranking www. aristotleprep. com 3 IntroductionA chance of GMAT instructors exhort that to improve their score on the Critical Reasoning section of the GMAT, students should pr spotice from LSAT Logical Reasoning skepticisms. While over wholly this st swangy is fine, atomic weigh 53 fuss students face is that a lot of the interrogatorys on the LSAT be non representative of those on the GMAT; nigh run through with(predicate) dubious attends while near(a) ar of a question type that is non tried on the GMAT at altogether. To booster students overcome this riddle we, at Aristotle, dupe come surface with compilation of hundred and superstar LSAT questions that ar rattling similar to those that students ar promising to see on the GMAT.All these questions select been compiled from last(prenominal) formal LSAT tests (tests which argon sound outily available in the in the national eye(predicate) do master(prenominal)) and we ch ch immerseern? t coerce whatsoever changes to these. An answer key has been provided at the end of this concurlet provided expla lands for sever in eachy(prenominal) answer bind non been provided. In case you want detailed expla republic to a question(s) please post the question(s) on „Forums? section on our website †www. aristotleprep. com †and adep t of experts bequeath revert within 24 hours of the same. Good luck! CR Pr flirtice Set †1 www. aristotleprep. com the the the our 4 1.Ann: All the campers at encamp Winnehatchee go to Tri -Cities High School Bill: That? s non true. near Tri-Cities students atomic play 18 campers at bivouacking Lakemont. Bill? s answer back be best explained on the boldness that he has stockn Ann? s remark to mean that (A) or so of the campers at Camp Lakemont come from spunky schools precedent(a)(a) than T riCities (B) close to Tri-Cities High School students be campers at Camp Winnehatchee (C) close to Tri-Cities High School students wipe out withdrawn from Camp Lakemont (D) totally Tri-Cities High School students engage withdrawn from Camp Lakemont (E) solo campers at Camp Winnehatchee atomic consider 18 students at Tri -Cities High School . More than a form ag bingle, the metropolis de n sensation that constabulary would crack stack on law slightly greennessi ng loted cars and that resources would be am bodily exertiond from writing fagnonball along tickets to ticketing illeg protagonist parked cars. still no crackdown has taken place. The police headman claims that resources take over had to be diverted from writing speeding tickets to combating the city? s swag treat problem. Yet the police argon quench writing as m whatsoever speeding tickets as ever. in that locationfore, the excuse nigh resources be tied up in fighting do drugs-related crime simply is non true.The destruction in the act depends on the supposal that (A) every atom of the police cast is adapted to drop dead on combating the city? s drug problem (B) drug-related crime is non as safe a problem for the city as the police chief claims it is (C) writing speeding tickets should be as measurable a priority for the city as combating drug-related crime (D) the police could be cracking down on il licitly parked cars and combating the drug problem withou t having to reduce writing speeding tickets (E) the police nooky non live writing as m either speeding tickets as ever while diverting resources to combating drug-related crime . desiccated sess clippings mixed into tend opinery in stages snap, providing food for thoughts for beneficial undercoat bacteria. This results in sm change -than- fair go under return. Yet mixing fresh git clippings into garden farming usually actors poorer than- tumefy(p) plant growth. Which unitary of the next, if true, close to helps to explain the difference in plant growth fall upond in a risque place? (A) The number of beneficial s anele bacteria ill-use-ups whenever either kind of plant fabric is mixed into garden s oil. (B) Nutrients released by dried grass clippings ar nowadays available to beneficial soil bacteria. ww. aristotleprep. com 5 (C) Some dried grass clippings curb nutrients originally derived from commercial lawn fertilizers, and therefore provide add itive enrichment to the soil. (D) Fresh grass clippings mixed into soil decompose rapidly, generating blue use ups of heat that kill beneficial soil bacteria. (E) When a mix of fresh and dried grass clippings is mixed into garden soil, plant growth oft times waterfall. 4. A gas assess of unrivaled cent per gallon would raise unrivaled billion dollars per class at menstruum consumption esteems.Since a revenue of cardinal cents per gallon would so raise fifty billion dollars per contrast of study, it seems a perfect bearing to bundle with the federal budget deficit. This value would hand over the addi tional expediency that the resulting drop in the demand for gasoline would be ecologically sound and would forbid our country from being too dependent on foreign oil prep argonrs. Which atomic number 53 of the undermenti stard nearly intelligibly identifies an fracture in the germ? s conclude? (A) The author advances irrelevant data. (B) The author relie s on monstrous current consumption figures. C) The author confuse gots incompatible trusts. (D) The author mistakes an final result for a cause. (E) The author costs to conscience rather than reason. 5. in that location is no reason why the hold out of scientists has to be officially confirmed in the first place being published. on that point is a dodging in place for the verification or disconfirmation of scientific finding, namely, the replication of results by early(a) scientists. Poor scientific work on the bind got out of every whiz scientist, which dope include everything from incautious reportage practices to fraud, is not harmful. It en dep whizz be undetermined and rendered harm little hen around other scientists conduct the experiments and stick disconfirmatory results. Which angiotensin converting enzyme of the sideline, if true, would weaken the line of reasoning? (A) Scientific experiments crapper go un quarreld for numerous eld forward they ar replicated. (B) Most scientists work in universities, where their work is submitted to peer review before payoff. (C) Most scientists argon under pressure to contact their work handy to the scrutiny of replication. (D) In scientific experiments, cargon little reporting is much common than fraud. (E) Most scientists work as part of a team rather than al cardinal. 6.Alice: Quotas on railroad car imports to the join States should be eliminated. Then national producers would beat to compete directly with Japanese manu concomitanturers and would be agonistic to produce higher-quality cars. Such competition would be unattackable for consumers. www. aristotleprep. com 6 David: You fail to realize, Alice, that quotas on automobile imports ar pervasive ballwide. Since German, Britain, and France afford quotas, so should the United States. Which champion of the hobby around blamelessly characterizes David? s response to Alice? s statement? (A) David falsely accuses Alice of contradicting herself. B) David un fair directs his cable against Alice soulfulnessally. (C) David uncovers a hidden assumption cardinal Alice? s position. (D) David takes a position that is similar to the one Alice has taken. (E) David fails to apostrophize the reasons Alice cites in sp ar of her determination. 7. Governments countenance except one response to public criticism of soci ally necessary services: regulation of the activity of providing those services. precisely governments inevitably require the activity to a great extent pricy by regulating it, and that is particularly trouble nearly in these ms of strained financial resources.However, sin ce public criticism of pip-squeak-cargon services has undermined all reliance in much(prenominal) services, and since such services atomic number 18 socially necessary, the government is current to move. Which one of the future(a) statements bed be sympathisered from the transportation? (A) The qual ity of peasant aid testament improve. (B) The cost of providing child-c atomic number 18 services will plus. (C) The government will use funding to encourage advances in child c be. (D) If public criticism of constitution is strongly voiced, the government is veritable to respond. (E) If child-cargon services are not regulated, the cost of providing child care will not increase. . Advertisers are often criticized for their unscrupulous usage of plenty? s tastes and wants. on that point is try, however, that some revealrs are motivated by moral as s intimately as financial considerations. A particular publication decided to change its image from being a family hotspaper to concentrating on sex and violence, thus challenge to a disparate readership. Some advertisers withdrew their advertisements from the publication, and this mustiness(prenominal) ready been because they morally disapproved of publishing salacious material. Which one of the following, if true, would virtually streng past the contestation? A) The advertisers switched their advertisements to other family peeledspapers. (B) Some advertisers switched from family newspapers to advertise in the changed publication. (C) The advertisers expected their ware sales to increase if they stayed with the changed publication, but to slack if they withdrew. (D) People who chiefly read family newspapers are not seeming to buy newspapers that tolerate on sex and violence. (E) It was expected that the changed publication would appeal principally to those in a antithetic income group. www. aristotleprep. com 7 9.If retail stores experience a decrease in r howeverues during this vacation season, then either attitudes toward extravagant gift- fine-looking develop changed or footings take uprise beyond the level nigh commonwealth quite a little afford. If attitudes have changed, then we all have something to celebrate this season. If prices have rise beyond the level roughly pile can afford, then it must be that salaries have not unplowed pace with rising prices during the past year. anticipate the premises higher up to be true, if salaries have kept pace with rising prices during the past year, which one of the following must be true? (A) Attitudes toward extravagant gift-giving have changed. B) Retail stores will not experience a decrease in retail sales during this vacation season. (C) Prices in retail stores have not risen beyond the level that roughly people can afford during this holiday season. (D) Attitudes toward extravagant gift-giving have not changed, and stores will not experience a decrease in revenues during this holiday season. (E) Either attitudes toward extravagant gift-giving have changed or prices have risen beyond the level that most people can afford during this holiday season. 10. The â€Å"suicide wave” that followed the United States rootage trade crash of October 1929 is more(prenominal) than legend than fact.Careful exa mination of the monthly figures on the causes of death in 1929 specifys that the number of suicides in October and in November was comparatively low. In exclusively if ternary other months were the monthly figures inflict. During the summer months, when the stock commercialise was flourishing, the number of suicides was substantially higher. Which one of the following, if true, would best challenge the conclusion of the hitage? (A) The suicide rate is influenced by legion(predicate) psychological, interpersonal, and societal factors during any aban affordd diachronic period. B) October and November have almost al foc employs had relatively high suicide pass judgment, even during the 1920s and 1930s. (C) The suicide rate in October and November of 1929 was considerably higher than the average for those months during several(prenominal)(prenominal) preceding and following geezerhood. (D) During the days surrounding the stock market crash, suicide place were typically lo wer at the beginning of any calendar year than toward the end of that year. (E) Because of seasonal differences, the number of suicides in October and November of 1929 would not be expected to be the same as those for other months. 11.L attracting how to build a nest p seculars an important part in the preparation mastery of birds. For example, Dr. atomic number 6 has recorded the success of a number of b missbirds in several successive years. He finds that birds nesting for the first quantify are less boffo in breeding than are older birds, and also less favored than they themselves are a year posterior. This cannot be a mere matter of size and strength, since blackbirds, like the grand studyity of birds, are fully grown when they put across www. aristotleprep. com 8 the nest. Thus, it is difficult to cancel the conclusion that they expediency by their nesting experience.Which one of the following, if true, would most weaken the motive? (A) Blackbirds build better ne sts than other birds. (B) The capacity of blackbirds to lay viable eggs increases with apiece successive tally during the first few years of reproduction. (C) The breeding success of birds nesting for the second time is greater than that of birds nesting for the first time. (D) shrimpyr and weaker blackbirds breed just as successfully as bigger and stronger blackbirds. (E) Up to 25 percent of all birds are killed by predators before they start to nest. 12.How do the airlines expect to prevent commercial plane crashes? Studies have demonstraten that voyage wrongdoing contributes to ii-thirds of all such crashes. To address this problem, the airlines have upgraded their formulation programs by change magnitude the hours of classroom instruction and emphasizing communication skills in the cockpit. hardly it is unrealistic to expect such measures to compensate for pilots? lack of actual leakage time. Therefore, the airlines should re retrieve their training cost to reducing commercial crashes. Which one of the following is an assumption upon which the billet depends? A) Training programs can eliminate pilot errors. (B) Commercial pilots routinely undergo additional training passim their careers. (C) The number of airline crashes will decrease if pilot training programs focus on increasing actual prompt time. (D) Lack of actual flying time is an important contributor to pilot error in commercial plane crashes. (E) Communication skills are not important to pilot training programs. Questions 13-14 are based on the following: Despite improvements in interference for asthma attack, the death rate from this infirmity has duple during the past decade from its previous rate.Two possible explanations for this increase have been offered. First, the arrangement of deaths callable to asthma has stupefy more widespread and accurate in the past decade than it had been antecedently. Second, there has been an increase in urban pollution. However, since the rate of deaths due to asthma has change magnitude dramatically even in cities with long-standing, comprehensive medical checkup records and with little or no urban pollution, one must instead conclude that the cause of increased deaths is the use of bronchial inhalers by asthma sufferers to relieve their symptoms. 13.Each of the following, if true, provides take hold to the strain EXCEPT: (A) Urban populations have cardinal-fold in the past decade. (B) Records of asthma deaths are as accurate for the past twenty years as for the past ten years. (C) Evidence suggests that bronchial inhalers befool the lungs more sensitive to irritation by airborne pollen. www. aristotleprep. com 9 (D) By temporarily relieving the symptoms of asthma, inhalers encourage sufferers to avoid more beneficial measures. (E) Ten years ago bronchial inhalers were not available as an asthma treatment. 14. Which one of the following is an assumption on which the argument depends? A) Urban pollution has n ot doubled in the past decade. (B) Doctors and patients generally ignore the role of allergies in asthma. (C) Bronchial inhalers are unsafe, even when utilize fit in to the recommended instructions. (D) The use of bronchial inhalers aggravates other diseases that often emit among asthma sufferers and that often lead to expandal outcomes even when the asthma itself does not. (E) Increased urban pollution, improved recording of asthma deaths, and the use of bronchial inhalers are the only possible explanations of the increased death rate due to asthma. 15.There is little point in looking to mechanics for insights into governmental issues. Most of them hold governmental views that are less insightful than those of any reasonably well improve person who is not an artist. Indeed, when taken as a whole, the statements make by artists, including those considered great, signalise that artistic genius and political insight are seldom build together. Which one of the following c an be inferred from the passage? (A) There are no artists who have insights into political issues. (B) A thorough education in art makers a person reasonably well educated. C) every(prenominal) reasonably well-educated person who is not an artist has more insight into political issues than any artist. (D) Politicians rarely have any artistic talent. (E) Some artists are no less politically insightful than some reasonably well educated persons who are not artists. 16. Rita: The original purpose of government farm bounty programs was to provide income st index for downhearted family farmers. But most farm -subsidy specie goes to a few farmers with astronomical holdings. Payments to farmers whose income, before subsidies, is greater than $ vitamin C,000 a year should be find outped.Thomas: It would be impossible to administer such a cut -off point. Subsidies are trained during the planting and developing season, but farmers do not k presently their income for perpetraten calen dar year until levy returns are cypher and submitted the following April. Which one of the following, if true, is the strongest counter Rita can make to Thomas remonstration? (A) It has choke difficult for small farmers to arrive bank loans to be repaid later by money from subsidies. (B) Having such a cut-off point would cause some farmers whose income would otherwise exceed $100,000 to reduce their plantings. www. ristotleprep. com 10 (C) The income of a farmer varies because weather and market prices are not perpetual from year to year. (D) If subsidy payments to considerable farmers were eliminated the financial condition of the government would improve. (E) Subsidy cut-offs can be determined on the hind end of income for the preceding year. 17. recent physicians often employee laboratory tests, in addition to somatic examinations, in raise to diagnose diseases accurately. Insurance confederation regulations that deny coverage for certain laboratory tests hence decr ease the quality of medical care provided to patients.Which one of the following is an assumption that would serve to justify the conclusion in a higher place? (A) Physical examinations and the uncovered laboratory tests together provide a more accurate diagnosing of many diseases than do somatogenic examinations alone. (B) some(prenominal) physicians generally oppose insurance company regulations that, in roam to reduce costs, limit the use of laboratory tests. (C) Many patients who might benefit from the uncovered laboratory tests do not have any form of wellness insurance. D) There are some illnesses that experienced physicians can diagnose accurately from physicians examination alone. (E) Laboratory tests are more costly to perform than are physical examinations. 18. Oil analysts predict that if the price of oil falls by half(a), the consumers purchase price for gasoline make from this oil will also fall by half. Which one of the following, if true, would cast the most dea r doubtfulness on the prediction made by the oil analysts? (A) Improved automobile technology and new kinds of dismiss for cars have enabled some drivers to use less gasoline. B) accelerator pedal manufacturers will not expand their profit margins. (C) There are many different gasoline companies that compete with each othe r to provide the most glossy price to consumers. (D) Studies in several countries show that the add of gasoline purchased by consumers initially rises by and by the price of gasoline has fallen. (E) Refining costs, distribution costs, and assesses, none of which varies importantly with oil prices, constitute a heavy(a) division of the prices of gasoline. 19. A take after was recently conducted among ferry passengers on the North Sea.Among the results was this: more of those who had taken anti -seasickness medicine before their slip-up reported symptoms of seasickness than those who had not taken such medication. It is clear then, that despite claims by drug companies that clinical tests show the adverse, people would be better off not taking anti -seasickness medications. Which one of the following, if true, would most weaken the conclusion preceding(prenominal)? www. aristotleprep. com 11 (A) Given rough enough weather, most ferry passengers will have some symptoms of seasickness. (B) The clinical tests reported by the drug companies were conducte d by the drug companies staffs. C) People who do not take anti -seasickness medication are just as seeming to respond to a survey on seasickness as people who do. (D) The seasickness symptoms of the people who took anti -seasickness medication would have been more repellent had they not taken the medication. (E) People who have worn-out(a) money on anti -seasickness medication are less likely to admit symptoms of seasickness than those who have not. 20. Economic considerations colour every aspect of outside(a) transaction, and nations are just like individuals in that the lender sets the foothold of its dealings with the geter.That is why a nation that owes money to anoth er nation cannot be globe leader. The reasoning in the passage assumes which one of the following? (A) A nation that does not lend to any other nation cannot be a world leader. (B) A nation that can set the call of its dealings with other nations is certain to be a world leader. (C) A nation that has the basis of its dealings with other action set by that nation cannot be a world leader. (D) A nation that is a world leader can borrow from another nation as long as that other nation does not set the terms of the dealings mingled with the devil nations. E) A nation that has no dealings with any other nation cannot be world leader. 21. Political theorist: The chief rigations of all governments are the court-ordered constitution and the police force and as there cannot be a good legal system where the police are not well paid, it follows that where the police are well paid there wi ll be good legal system. The reasoning in the argument is not sound because it fails to establish that: (A) many governments with regretful legal systems have bad paid police forces. B) bad governments with good legal systems must have poorly paid police forces. (C) a well-paid police force cannot be effective without a good legal system. (D) a well-paid police force is sufficient to plug a good legal system (E) some bad governments have good legal systems. 22. judicial system records from medieval France show that in the years 1300 to 1400 the number of people arrested in the french realm for â€Å" uncultivated interpersonal crimes” (not committed in wars) increased by 30 percent over the number of people arrested for such crimes in the years 1200 to 1300.The increase was not the result of false arrests; therefore, medieval France had a higher level of record interpersonal violence in the years 1300 to 1400 than in the years 1200 to 1300. Which one of the following s tatements, if true, most seriously weakens the argument? www. aristotleprep. com 12 (A) In the years 1300 to 1400 the French governments category of violent crimes include an increasing variety of interpersonal crimes that are genuinely nonviolent. (B) Historical accounts by monastic chroniclers in the years 1300 to 1400 are fill ed with descriptions of violent attacks committed by people victuals in the French realm C) The number of individual agreements between two people in which they swore oaths not to attack each other increased substantially later 1300. (D) When position armies tried to conquer parts of France in the middle †to late 1300s. violence in the northern province of Normandy and the south -western province of Gascony increased. (E) The population of medical France increased substantially during the first five decades of the 1300s until the deadly bubonic plague decimated the population of France after 1348. 23.Rhizobium bacteria living in the roots of bean plants or other le gumes produce fixed north which is one of the essential plant nutrients and which for non -legume crops, such as shuck normally must be supplied by applications of nitrogen -based fertilizer. So if biotechnology succeeds in producing wheat strains whose r oots will play host to Rhizobium bacteria, the need for artificial fertilizers will be reduced. The argument in a higher place makes which one of the following assumptions? (A) Biotechnology should be tell toward producing plants that do not require artificial fertilizer. B) better nitrogen is currently the only soil nutrient that must be supplied by artificial fertilizer for growing wheat crops. (C) There are no naturally occurring strains of wheat or other grasses that have Rhizobium bacteria living in their roots. (D) Legumes are currently the only crops that produce their own grant of fixed nitrogen. (E) Rhizobium bacteria living in the roots of wheat would produce fixed nitrogen. 24. authoritative leg islation that requires designated sections for flockrs and non-smokers on the premises of closed-doorly own billetes is an intrusion into the private sector that cannot be justified.The fact that studies indicate that non-smokers might be harmed by inhaling the smoke from others cig arettes is not the main issue. Rather, the main issue concerns the governments rape of the reclaim of private businesses to determine their own policies and rule. Which one of the following is principle that, if doned, could enable the conclusion to be decently drawn? (A) Government intrusion into the policies and rules of private businesses is justified only when individuals might be harmed. (B) The right of individuals to breathe safe air supersedes the right of businesses to be free from government intrusion. C) The right of businesses to self-determination overrides whatever right or duty the government may have to cling to the individual. (D) It is the duty of private businesses to protec t employees from harm in the workplace. (E) Where the rights of businesses and the duty of government conflict, the main issue is finding a successful compromise. www. aristotleprep. com 13 25. Leachate is a solution, frequently passing contaminated, that develops when water permeates a landfill site. If and only if the landfills capacity to hold liquids is exceeded does the leachate bleed into the surround, genera lly in unpredictable quantities.A method must be found for disposing of leachate. Most landfill leachate is send directly to sewage treatment plants, but not all sewage plants are equal to(p) of handling the highly contaminated water. Which one of the following can be inferred from the passage? (A) The ability to predict the volume of escaping landfill leachate would help solve the electric pig problem. (B) If any water permeates a landfill, leachate will escape into the environment. (C) No sewage treatment plants are capable of handling leachate. (D) Some landfill lea chate is send to sewage treatment plants that re incapable of handling it. (E) If leachate does not escape from a landfill into the environment, then the landfills capacity to hold liquids has not been exceeded. 26. The soaring prices of scholarly and scientific diarys have forced academic libraries employ only by academic enquiryers to drastically reduce their list of subscriptions. Some have suggested that in each academic discipline subscription decisions should be determined solely by a journals service program in that discipline, measured by the frequency with which it is cited in published writings by searchers in the discipline.Which one of the following, if true, most seriously calls into question the sugge stion described above? (A) The non-academic readership of a scholarly or scientific journal can be accurately gauged by the number of quantify articles appearing in it are cited in daily newspapers and popular magazines. (B) The average distance of a journal article in some sciences, such as physics, is less than half the average length of a journal article in some other academic disciplines, such as history. (C) The more and more expensive scholarly journals are less and less likely to be available to the general public from non-academic public libraries. D) Researchers often will not cite a journal article that has influenced their work if they think that the journal in which it appears is not highly regarded by the leading researchers in the mainstream of the discipline (E) In some academic disciplines, controversies which begin in the pages of one journal spill over into articles in other journals that are widely read by researchers in the discipline. 27. The average level of fat in the blood of peo ple crucifixion from subchills and fever cases of disease W is lower than the average level for the population as a whole.Nevertheless, most doctors believe that reducing blood-fat levels is an effective way of preventing acute W. www. arist otleprep. com 14 Which one of the following, if true, does most to justify this apparently paradoxical belief? (A) The blood level of fat for patients who have been cured of W is on average the same as that for the population at large. (B) several(prenominal) of the symptoms feature of acute W have been produced in laboratory animals fed large doses of a synthetic substance fat substitute, though acute W itself has not been produced in this way. C) The comeion from latent to acute W can occur only when the agent that causes acute W absorbs large quantities of fat from the patients blood. (D) The levels of fat in the blood of patients who have disease W respond abnormally slowly to changes in dietetical expenditure of fat. (E) High levels of fat in the blood are indicative of several diseases that are just as serious as W. 28. Baking for winter holidays is customs duty that may have a sound medical basis. In midwinter, when days are short, many people suffer from a specific ty pe of seasonal depression caused by lack of sunlight.Carbohydrates, both sugars and starches, raise the brains levels of serotonin, a neurotransmitter that improve the mood. In this respect, carbon hydrates act on the brain in the same way as some antidepressants. Thus, eating holiday cookies may provide an effective form of self incontrovertible medication. Which one of the following can be justly inferred from the passage? (A) Seasonal depression is one of the most easily treated forms of depression. (B) Lack of sunlight lowers the level of serotonin in the brain. (C) People are more likely to be depressed in midwinter than at other times of the year. D) Some antidepressants act by changing the brains level of serotonin. (E) Raising the level of neurotransmitters in the brain effectively relieves depression. 29. The current intent to give college students a broader choice in planning their own courses of study should be abandoned. The students who are supporting the proposal will never be squelched, no matter what indispensablenesss are established. Some of these students have reached their third year without declaring a major. One first-year student has failed to arrant(a) quaternity demand courses. Several others have indicated a serious indifference to grades and intellectual achievement.A mar in the argument is that it does which one of the following? (A) avoids the issue by foc employ on supporters of the proposal (B) argues circularly by assuming the conclusion is true in stating the premises. (C) fails to find out the critical term â€Å"satisfied” (D) distorts the proposal advocated by opponents (E) users the term â€Å"student” equivocally 30. The question whether reasoning(a) life exists elsewhere in the cosmos is for sure imprecise because we are not sure how different from us something might be and still count as â€Å" able life”. Yet we cannot just decide to define â€Å" happy life” in www. ristotlepre p. com 15 some more precise way since it is likely that we will find and recognize intelligent life elsewhere in the universe only if we leave our definitions open to new, unimagined possibilities. The argument can most reasonably be interpreted as an objection to which one of the following claims? (A) The question whether intelligent life exists elsewhere in the universe is one that will never be correctly answered. (B) Whether or not there is intelligent life elsewhere in the universe, our understanding of intelligent life is limited. (C) The question nearly the beingness of intelligent life elsewhere in the niverse must be made more precise if we hope to answer it correctly. (D) The question whether there is intelligent life elsewhere in the universe is so imprecise as to be meaningless. (E) The question whether there is intelligent life elsewhere in the universe is one we should not spend our time trying to answer. 31. Pedro: Unlike cloth nappys, usable diapers are a threat to the environment. Sixteen billion fluid diapers are discarded annually, filling up landfills at an alarming rate. So people must s transgress buying disposable diapers and use cloth diapers.Maria: But you forget that cloth diapers must be rinse in hot water, which requires energy. Moreover, the resulting wastewater tarnishs our rivers. When families use diaper services, diapers must be delivered by fuel -burning trucks that pollute the air and add to traffic congestion. Maria objects to Pedros argument by (A) claiming that Pedro overstates the negative evidence most(predicate) disposable diapers in the course of his argument in favour of cloth diapers. (B) indicating that Pedro draws a hasty conclusion, based on inadequate evidence well-nigh cloth diapers. C) pointing out that there is an ambiguous use of the word â€Å"disposable” in Pedros argument (D) demonstrating that cloth diapers are a far more serious threat to the environment than disposable diapers are (E) su ggesting that the frugal advantages of cloth diapers outmatch whatever environmental defile they may cause 32. In an experiment, two-year-old boys and their grows made pie dough together using ringlet pins and other utensils. Each father-son bridge used a rolling pin that was distinctively different from those used by the other, â€Å"father -son pairs, and each father repeated the phrase â€Å"rolling pin” each time his son used it.But when the children were asked to bring out all of the rolling pins among a group of kitchen utensils that included several rolling pins, each child picked only the one that he had used. Which one of the following inferences is most back up by the information above? (A) The children did not nab the function of rolling pin. www. aristotleprep. com 16 (B) No two children understood the name â€Å"rolling pin” to moderate to the same object (C) The children understood that all rolling pins have the same general shape. (D) Each chi ld was able to identify correctly only the utensils that he had used. E) The children were not able to distinguish the rolling pins they used from other rolling pins. 33. When 100 people who have not used cocain are tested for cocaine use, on average only 5 will test positive. By contrast, of every 100 people who have used cocaine 99 will test positive. Thus, when a randomly chosen group of peopl e is tested for cocaine use, the grand majority of those who test positive will be people who have used cocaine. A reasoning error in the argument is that the argument (A) attempts to infer a value judgment from purely existent premises. B) attributes to every member of the population the properties of the average member of the population. (C) fails to take into account what proportion of the population have used cocaine. (D) ignores the fact that some cocaine users do not test positive. (E) advocates testing people for cocaine use when there is no reason to leery that they have used c ocaine. 34. With the passage of the new tax enlighten law the annual tax lading on low -income taxpayers will be reduced, on average, by anywhere from $100 to $300. Clearly, tax reform is in the come to of low-income taxpayers.Which one of the following, if true, most undermines the conclusion above? (A) Tax reform, by simplifying the tax calculate will save many people the expense of having an accountant do their taxes. (B) Tax reform, by eliminating tax incentives to build rental housing, will push up rents an average of about $40 per month for low-income taxpayers. (C) Low-income taxpayers have consistently voted for those political candidates who are strong advocates of tax reform. (D) The new tax reform laws will licence low and middle-income taxpayers to deduct Child-care expenses from their taxes. E) Under the new tax reform laws, many low-income taxpayers who directly pay taxes will no Longer be required to do so. 35. If we are to expand the exploration of our solar sy stem our next manned flight should be to Phobos, one of Mars? s moons, rather than to Mars itself. The fli ght times to each are the same but the Phobos journey would require less than half the fuel load of a Mars expedition and would, therefore, be much less costly. So, it is clear that Phobos should be our next step in space exploration. Which one of the following, if true, would most help to explain the difference in fuel requirement? A) More equipment would be required to explore Phobos than to explore Mars. (B) Smaller spaceships require less fuel than bigger spaceships. www. aristotleprep. com 17 (C) Information well-educated during the turn on to Phobos can be used during a subsequent trip to Mars. (D) The shortest distance between Phobos and Mars is less than half the shortest distance Between Earth and Mars. (E) Lift-off for the return trip from Phobos requires much less fuel than that from Mars because of Phobos weaker gravitational pull. 36.Scientific research that in volves internationalistic coaction has produ ced papers of greater influence, as measured by the number of times a paper is cited in subsequent papers, than has research without any collaboration. Papers that result from international collaboration are cited an average of seven times, whereas papers with superstar authors are cited only three times on average. This difference shows that research projects conducted by international research teams are of greater importance than those conducted by champion researchers. Which one of the following is an assumption on which the argument depends? A) Prolific sources can inflate the number of citations they suck in by citing themselves in subsequent papers. (B) It is possible to understand whether or not a paper is the product of international collaboration by determining the number of citations it has received (C) The number of citations a paper receives is a measure of the importance of the research it reports. (D) The collaborativ e efforts of scientists who are citizens of the same country do not produce papers that are as important as papers that are prod uced by international collaboration. E) International research teams tend to be more generously funded than are single researchers. 37. It is more suited to have some form of socialized medicine than a system of medical care relying on the private sector. Socialized medicine is more slackly accessible than is private-sector system. In addition, since countries with socialized medicine have a lower infant deathrate rate than do countries with a system relying entirely on the private sector, socialized medicine seems to be technologically superior.Which one of the following best indicates a flaw in the argument about the technological favourable position of socialized medicine? (A) The lower infant mortality rate might be due to the systems allowing greater access to Medical care (B) There is no necessary connection between the economic system of sociali sm and Technological achievement. (C) Infant mortality is a reliable indicator of the quality of medical care for children. (D) No list is presented of the countries whose infant mortality statistics are summarized under the two categories, â€Å"socialized” and â€Å"private-sector”. E) The argument presupposes the desirability of socialized medicine, which is what the Argument seeks to-establish. www. aristotleprep. com 18 38. Lourdes: Dietary type is an important part of a healthful diet. Experts recommend that adults consume 20 to 35 grams of graphic symbol a day. Kyra: But a daily intake of eccentric that is significantly above that recommended level interferes with mineral absorption, specially the absorption of calcium. The public should be told to cut act on fibre intake Which one of the following, if true, most undermines Kyra? s recommendation? A) Among adults, the average consumption of dietary fibre is at present approximately 10 grams a day. (B) The m ore a food is processed, the more the fibre is broken down and the lower the fibre content. (C) Many foodstuffs that are excellent sources of fibre are economical and readily available (D) Adequate calcium intake helps prevent the decrease in lift mass cognize as osteoporosis. (E) Many foodstuffs that are excellent sources of fibre are popular with consumers. 39. intensity Review: When I read a unfermented set in a city I k promptly well, I must see that the author knows the city as well as I do if I am to take that writer seriously.If the writer is faking I know immediately and do not trust the writer. When a novelist demonstrates the required knowledge, I trust the story teller, so I trust the tale. This trust increases my enjoyment of a good novel. scratch leewards second novel is set in San Francisco, in this novel, as in his first, Lee passes my test with flying colours. Which one of the following can be properly inferred from the passage? (A) The book observer enjoys vi rtually any novel written by a novelist whom she trusts B) If the book reviewer trusts the novelist as a storyteller, the novel in question must be set in a city the book reviewer knows well (C) Peter Lees first novel was set in San Francisco (D) The book reviewer does not trust any novel set in a city that she does not know well (E) The book reviewer does not believe that she knows San Francisco better than Peter Lee does 40. Prominent business executive directors often play ready roles in United States presidential campaigns as fundraisers or backroom strategists but few actually seek to kick the bucket president themselves.Throughout history the great majority of those who have sought to become president have been l awyers, military leaders, or full -time politicians. This is understandable, for the personality and skills that make for success in business do not make for success in politics. Business is generally hierarchical, whereas politics is coordinative; as a result, b usiness executives tend to be un cozy with compromises and power sharin g, which are inherent in politics. Which one of the following, if true, most seriously weakens the proposed explanation of why business executives do not run for president? A) Many of the most active voice presidential fundraisers and backroom strategists are themselves politicians. www. aristotleprep. com 19 (B) Military leaders are generally no more comfortable with compromises and power sharing than are business executives. (C) Some of the skills needed to become a successful lawyer are different from some of those needed to become a successful military leader. (D) Some former presidents have engaged in business ventures after leaving office (E) Some hierarchically organize companies have been major financial supporters of candidates for president. 1. A scientific guess is a good theory i f it satisfies two requirements †it must accurately describe a large class of observations in terms of a stick tha t is simple enough to defend only a few elements, and it must make definite predictions about the results of future observations. For example, Aristotle? s cosmogonic theory, which claimed that everything was made out of four elements †earth, air, fire, and water †satisfied the first requirement but it did not make any definite prediction. Thus, Aristotle? cosmological theory was not a good theory. If all the statements in the passage are true, each of the following must also be true EXCEPT: (A) Prediction about the results of future observations must be made by any good scientific theory. (B) Observation of physical phenomena was not a major concern in Aristotle? s cosmological Theory (C) Four elements can be the basis of a scientific pose that is simp le enough to meet the Simplicity cadence of a good theory. (D) A scientific model that contains many elements is not a good theory (E) Aristotle? cosmological theory described a large class of observations in terms of o nly four elements. 42. Compared to non-profit hospitals of the same size, investor-owned hospitals require less public investment in the form of tax breaks, use few employees, and have higher occupancy levels. It can therefore be concluded that investor -owned hospitals are a better way of delivering medical care than are non-profit hospitals. Which one of the following, if true, most undermines the conclusion drawn above? (A) Non-profit hospitals aim more per bed than do investor-owned hospitals. B) Patients in non-profit hospitals rectify more quickly than don patients with comparable Illnesses in investor-owned hospitals (C) Non-profit hospitals do more fund fosterage than do investor-owned hospitals. (D) Doctors at non-profit hospitals earn higher salaries than do similarly qualified doctors at investor-owned hospitals. (E) Non-profit hospitals receive more donations than do investor-owned hospitals. 43. The ancient Egyptian pharaoh Akhenaten, who had a profound effect durin g his lifetime on Egyptian art and religi on, was well loved and highly respected by his subjects.We know this from the fierce allegiance show to him by his palace guards, as documented in reports written during Akhenaten? s reign. A dubitable technique used in the argument is to: www. aristotleprep. com 20 (A) Introduce information that actually contradicts the conclusion (B) imprecate on evidence that in principle would be impossible to challenge (C) Make a set out based on a sample that is likely to be unrepresentative (D) Depend on the equivocalness of the term â€Å"ancient” (E) Apply present-day standards in an inappropriate way to ancient times 44.Zelda: Dr. Ladlow, a research psychologist, has convincingly demonstrated that his theory about the determinants of rat behaviour generates consistently accurate predictions about how rats will perform in a internal ear. On the basis of this evidence Dr. Ladlow has claimed that his theory is irrefutably correct. Anson: T hen Dr. Ladlow is not responsible psychologist. Dr. Ladlow? s evidence does not once and for all prove that his theory is correct. Responsible psychologists always accept the first step that new evidence will show that their theories are incorrect.Which one of the following can be properly inferred from Anson? s argument? (A) Dr. Ladlow? s evidence that his theory generates consistently accurate predictions about how rates will perform in a maze is inaccurate (B) Psychologists who can derive consistently accurate predictions about how rats will perform in a maze from their theories cannot responsibly co nclude that those theories cannot be disproved (C) No matter how responsible psychologists are, they can never develop correct theoretical explanations. (D) Responsible psychologists do not make predictions about how rats will perform in a maze E) Psychologists who accept the possibility that new evidence will show that their theories are incorrect are responsible psychologists. 45. Smith: Meat in the diet is healthy, despite what some people say. after(prenominal) all, most doctors do eat meat, and who knows more about health than doctors do? Which one of the following is a flaw in Smith? s reasoning? (A) Attacking the opponents? motives instead of their argument (B) Generalizing on the basis of a sample consisting of a typical cases (C) presumptuous at the outset what the argument claims to establish through reasoning D) Appealing to authority, even when different government activity give conflicting advic e about an issue (E) Taking for granted that experts do not act counter to what, according to their expertise, in their best interest 46. The brains of monovular check are genetically identical. When only one of a pair of identical twins is a schizophrenic, certain areas of the affected twin? s brain are smaller than corresponding areas in the brain of the unaffected twin. No such differences are found when neither twin is schizophrenic. Therefore, thi s discovery www. aristotleprep. com 1 provides definitive evidence that schizophrenia is caused by damage to the physical structure of the brain. Which one of the following is an assumption required by the argument? (A) The brain of person suffering from schizophrenia is smaller than the brain of anyone not suffering from schizophrenia (B) The relative smallness of certain parts of the brains of schizophrenics is not the result of schizophrenia or of medications used i n its treatment. (C) The brain of a person with an identical twin is no smaller, on average, than the brain of person who is not twines. D) When a pair of identical twins both suffer from schizophrenia, their brains are the same size (E) People who have an identical twin are no more likely to suffer from schizophrenia than those who do not. 47. Sixty adults were asked to keep a diary of their meals, including what they consumed, when, and in the company of how many people . It was found that at meals with which they d graze alky beverageic beverages, they consumed about 175 calories more from non-alcoholic source than they did at meals with which they did not drink alcoholic beverages.Each of the following, if true, contributes to an explanation of the difference in caloric intake EXCEPT: (A) Diners spent a much longer time at meals served with alcohol than they did at those serve without alcohol. (B) The meals eaten later in the day tended to be bigger than those eaten to begin with in the day, and later meals were more likely to include alcohol. (C) People eat more when there are more people present at the meal, and more people tended to be present at meal served with alcohol than at meal s served without alcohol. D) The meals that were most carefully nimble and most attractively served tended to be those at which alcoholic beverages were consumed (E) At meals that included alcohol, relatively more of the agree calories consumed came from carbohydrates and relatively fewer of them came from fats and proteins. 48. Something must be done to ease traffic congestion. In traditional small towns, people used to work and shop in the same town in which they lived, but now that stores and workplaces are located far away from residential areas, people cannot avoid travelling long distances each day.Traffic congestion is so heavy on all roads that, even on major passs, the upper limit speed averages only 35 miles per hour. Which one of the following proposals is most supported by the statements above? (A) The maximum speed limit on major highways should be increased. (B) People who now travel on major highways should be support to travel on substitute roads instead. (C) Residents of the remaining traditional small towns should be encouraged to move to the suburbs. www. aristotleprep. com 22 (D) Drivers who travel well below the maximum speed limit on major highways should be fined. E) New businesses should be encouraged to locate closer to w here their workers would live. 49 . College professor: College students do not write nearly as well as they used to. about all of the papers that my students have done for me this year have been poorly written and ungrammatical. Which one of the following is the most serious weakness in the argument made by the professor? (A) It requires confirmation that the change in the professors students is representative of a change among college students in general. (B) It offers no proof to the effect that the professor is an accurate appraise of writing ability. C) It does not take into account the possibility that the professor is a poor teacher. (D) It fails to present contrary evidence. (E) It fails to define its terms sufficiently. 50. Mayor of Plainsville: In order to help the economy of Plainsville, I am using some of our tax revenues to help bring a major highway through the town and thereby attract new business to Plainsville. Citizens group: You must have interests other than our economy in mind. If you were si ncerely interested in helping our economy, you would instead allot the revenues to building a new business park since it would bring in twice the business that your highway would.The argument by the citizens group relies on which one of the following assumptions? (A) Plainsville presently has no major highways tally through it. (B) The mayor accepts that a new business park would bring in more new business than would the new highway. (C) The new highway would have no benefits for Plainsville other than attracting new business. (D) The mayor is required to get approval for all tax revenue allocation plans from the city council. (E) Plainsvilles economy will not be helped unless a new business park of the sort envisioned by the citizens group is built. 51.Recently, highly complete workers in easterly Europe have left jobs in record numbers to emigrate to the West. It is therefore likely that skilled workers who remain in Eastern Europe are in high demand in their home countries. Wh ich one of the following, if true, most seriously weakens the argument? (A) Eastern European factories select to hire workers from their home countries rather than to import workers from abroad. www. aristotleprep. com 23 (B) Major changes in Eastern European economic structures have led to the elimination of many positions previously held by the highly skilled emigrants. C) Many Eastern European emigrants need to acquire new skills after finding work in the West. (D) Eastern European countries plan to train many new workers to counterchange the highly skilled workers who have emigrated. (E) Because of the departure of skilled workers from Eastern European countries, many positions are now unfilled. 52. Two palaeontologists, Dr Tyson and Dr. Rees, disagree over the interpretation of certain footprints that were left among other footprints in hardened volcanic ash at site G. Dr.Tyson claims they are clearly early hominid footprints since they show military personnel characteristics : a squarish heel and a big toe immediately adjacent to the next toe. However, since the footprints indicate that if hominids made those prints they would have had to walk in an unexpected cross-stepping manner, by placing the left foot to the right of the right foot. Dr. Rees rejects Dr. Tysons conclusion. Which one of the following, if true, most seriously undermines Dr. Tysons conclusion? (A) The foot prints exhibit human characteristics were clearly those of at least two distinct individuals. B) Certain species of bears had feet very like human feet, except that the outside toe on each foot was the biggest toe and the innermost toe was the smallest toe. (C) Footprints regulate like a humans that do not show a cross -stepping pattern exist at site M, which is a mile away from site G, and the two sets of footprints are contemporaneous. (D) When the moist volcanic ash became sealed under additional layers of ash before hardening, some details of some of the footprints were erased . (E) Most of the other footprints at site G were of animals with hooves. 53.It is not know whether bovine spongiform encephalopathy (BSE), a disease of oxen invariably deadly to them, can be genetical directly from one give animal to another at all stages of the infection. If it can be, there is now a reservoir of infected cattle incubating the disease. There are no diagnostic tests to identify infected animals before the animals show overt symptoms. Therefore, if such direct transmission occurs, the disease cannot be eradicated by ____ Which one of the following best completes the argument? (A) removing from the herd and destroying any diseased animal as soon as it shows the typical symptoms of advanced BSE B) developing a drug that kills the agent that cause BSE, and then treating with that drug all cattle that might have the disease (C) destroying all cattle in areas where BSE occurs and raising cattle only in areas to which BSE is cognize not to have spread (D) developing a vaccine that confers lifelong immunity against BSE and giving it to all cattle, destroying in due course all those animals for which the vaccine protection came too late www. aristotleprep. com 24 (E) developing a diagnostic test that does identify any infected animal and destroying all animals found to be infected 4. Auto industry executive: Statistics show that cars that were built smaller af ter 1977 to make them more fuel-efficient had a higher incidence of accident-related fatalities than did their earlier larger counterparts. For this reason we oppose recent guidelines that would require us to produce cars with higher fuel efficiency. Which of the following, if true, would constitute the strongest objection to the executives argument? (A) Even after 1977, large automobiles were frequently involved in accidents that caused death or serious injury. B) Although fatalities in accidents involving small cars have increased since 1977, the number of accidents has decreased. (C) New c omputerized fuel systems can enable large cars to meet fuel efficiency standards established by the recent guidelines. (D) Modern technology can make small cars more fuel-efficient today than at any other time in their production history. (E) render efficiency in models of large cars rose immediately after 1977 but has been declining ever since. 55. No one who lacks knowledge of a subject is able to pass judgment on that subject.Since political know-how is a matter, not of adhering to technical rules, but of insight and style learned through apprenticeship and experience, only seasoned politicians are competent to judge whether a particular political form _or_ system of government is fair to all. A major weakness of the argument is that it (A) relies on a generalization about the characteristic that makes someone competent to pass judgment (B) fails to give specific examples to illustrate how political know-how can be acquired (C) uses the term â€Å"apprenticeship” to des cribe what is seldom a formalised relationship D) equates political know-how with understanding the social implications of political policies (E) assumes that when inexperienced politicians set policy they are control by the advice of more experienced politicians 56. Impact craters caused by meteorites smashing into earth, have been found all around the globe but they have been found in the greatest density in geologically changeless regions. This relatively greater abundance of securely identify crater s in geologically stable regions must be explained by the lower rates of d estructive geophysical processes in those regions.The conclusion is properly drawn if which one of the following is assumed? (A) A meteorite that strikes exactly the same spot as an earlier meteorite will obliterate all traces of the earlier impact. www. aristotleprep. com 25 (B) Rates of destructive geophysical processes within any given region vary markedly throughout geological time. (C) The rate at whic h the Earth is strike by meteorites has greatly increased in geologically recent times. (D) Actual meteorite impacts have been scattered fairly evenly over the Earths surface in the course of Earths geological history. E) The Earths geologically stable regions have been examine more intensively by geologists than have its less stable regions. 57. That the policy of atomic deterrence has worked thus far is unquestionable. Since the end of the Second human beings War, the very fact that there were nuclear armaments in existence has kept major powers from using nuclear weapons, for fright of starting a worldwide nuclear alternate that would make the land of the power initiating it uninhabitable. The proof is that a third world war between superpowers has not chanceed. Which one of the following, if true, indicates a flaw in the argument? A) Maintaining a high level of nuclear armaments represents a significant drain on a countrys economy. (B) From what has happened in the past, it is impossible to infer with certainty what will happen in the future, so an accident could still trigger a third world war between superpowers. (C) Continuing to produce nuclear weapons beyond the borderline needed for deterrence increases the likelihood of a nuclear accident. (D) The major powers have engaged in many smaller-scale military operations since the end of the Second World War, while refraining from a nuclear onfrontation. (E) It cannot be known whether it was nuclear deterrence that worked, or some other factor, such as a recognition of the economic value of remaining at peace. 58. A survey of alumni of the class of 1960 at Aurora University yielded puzzling results. When asked to indicate their academic rank, half of the respondents reported that they were in the top quarter of the graduating class in 1960. Which one of the following most helps account for the apparent contradict ion above? (A) A disproportionately large number of high -ranking alumni responded to the survey. B) Few, if any, respondents were mistaken about their class rank. (C) Not all the alumni who were actually in the top quarter responded to the survey. (D) Almost all of the alumni who graduated in 1960 responded to the survey. (E) Academic rank at Aurora University was based on a number of considerations in addition to average grades. 59. The ordinal-century physicist Sir Isaac due north is remembered chiefly for his treaties on motion and somberness. But north also conducted experiments secretly for www. aristotleprep. com 26 any years based on the arcane theories of alchemy, trying unsuccessfully to transmute common metals into gold and produce rejuvenating elixirs. If the alchemists of the seventeenth century had published the results of their experiments, chemistry in the eighteenth century would have been more advanced that it actually was. Which one of the following assumptions would allow the conclusion concerning eighteenth-century chemistry to be properly d rawn? (A) Scientific progress is retarded by the reluctance of historians to acknowledge the failures of some of the great scientists. B) Advances in science are hastened when reports of experiments, whether successful or not, are available for review by other scientists. (C) Newtons work on motion and gravity would not have gained wide acceptance if the results of his work in alchemy had also been made public. (D) change magnitude specialization within the sciences makes it difficult for scientists in one field to understand the principles of other fields. (E) The seventeenth-century alchemists could have achieved their goals only if their experiments had been subjected to public scrutiny. 0. Sedimentary rock hardens within the earths incrustation as lavers of matter accumulate and the pressure of the layers above converts the layers below into rock. One particular layer of aqueous rock that contains an unusual amount of the element atomic number 77 has been presented as support for a theory that a meteorite collided with the earth some sixty million years ago. Meteorites are rich in iridium compared to the earths crust, and geologists theorise that a meteorites collision with the earth raised a huge cloud of iridium-laden dust.The dust, they say, event\r\n'